mahanmath

New Member
ارسال ها
898
لایک ها
701
امتیاز
0
پاسخ : ماراتن ترکیبیات

مشکل ایکه تو. تایین نکردی کران بالا میخوای یا کران پایین.
نمیدونم چرا ولی* حس کردم کران پائین واسه مساله اصلا ارزش*ای نداره ! واسه همین نگفتم کران بالا می*خوام ...

حالا ولش کن یه سوال بذار جو عوض بشه .:35:
 

mojtabaaa1373

Active Member
ارسال ها
362
لایک ها
74
امتیاز
28
پاسخ : ماراتن ترکیبیات

یه سوال مرد پسند سوال 4 کامپیوتر دو سال پیش وقتی اول بودیم.سوال مر یم و مینا.
 

goodarz

Well-Known Member
ارسال ها
1,026
لایک ها
1,120
امتیاز
113
پاسخ : ماراتن ترکیبیات

نمیدونم چرا ولی* حس کردم کران پائین واسه مساله اصلا ارزش*ای نداره ! واسه همین نگفتم کران بالا می*خوام ...

حالا ولش کن یه سوال بذار جو عوض بشه .:35:
من با همون روشم میتونم یه کران بالا هم واسه سوال پیدا کنم, ولی ترجیح میدم این کارو نکنم, چون قیافش از این خیلی زشت تر میشه, بیخیال.....

مجتبی میشه بگی صورت سوال چیه؟
 

mojtabaaa1373

Active Member
ارسال ها
362
لایک ها
74
امتیاز
28
پاسخ : ماراتن ترکیبیات

یه درخت داریم دو نفر میخوان با هم بازی کنن اولی در صورتی میبره که همه ی رئوس گراف رنگ شن و دومی در غیر این صورت.
بازی:
6تا رنگ داریم هر بار میونیم یه راس رو رنگ کنیم به طوری که رنگش با همه ی مجاوراش تفاوت داشته باشه
ثابت کنید اولی استراتژی برد دارد.
من وقتی حلش کردم در پوست خودم نمی گنجیدم.:6:شرط میبندم شما هم در پوست خودتون نمی گنجید.:116:
 
ارسال ها
199
لایک ها
268
امتیاز
0
پاسخ : ماراتن ترکیبیات

مجتبی ببخشید. این سوال رو تقریبا هممون قبلا حل کردیم. اگه میشه تو ماراتن ها سوالاتی بگذارید که احتمال می دهید بقیه ندیده باشند. به نظر من هدف ماراتن ها آشنا شدن با سوالاتی است که فقط عده ی مخصوصی می دانند. مثلا سوالی که فقط معلم خاصی در کلاسش گفته یا سوالی که در یک وبلاگ گمنام گذاشته شده یا سوالی در یک کتاب نایاب و... .
 
C

counterexample

Guest
پاسخ : ماراتن ترکیبیات

یه درخت داریم دو نفر میخوان با هم بازی کنن اولی در صورتی میبره که همه ی رئوس گراف رنگ شن و دومی در غیر این صورت.
بازی:
6تا رنگ داریم هر بار میونیم یه راس رو رنگ کنیم به طوری که رنگش با همه ی مجاوراش تفاوت داشته باشه
ثابت کنید اولی استراتژی برد دارد.
من وقتی حلش کردم در پوست خودم نمی گنجیدم.:6:شرط میبندم شما هم در پوست خودتون نمی گنجید.:116:
این که راحته!
به نظرتون میشه استراتژی برد رو پیدا کرد؟
 

mojtabaaa1373

Active Member
ارسال ها
362
لایک ها
74
امتیاز
28
پاسخ : ماراتن ترکیبیات

امیدوارم این یکی رو تعداد زیادی ندیده باشند.سوال آسونیه ولی با مزه هست.
Suppose that every integer has been given one of the colours red, blue, green or yellow. Let
and
be odd integers so that
. Show that there are two integers of the same colour whose difference has one of the following values:
or
 

Kavoshgar

New Member
ارسال ها
397
لایک ها
479
امتیاز
0
سوالی از ترکیبیات

مجموع اعداد یک جدول ضرب n*n چیست ؟
 

rezashiri

Well-Known Member
ارسال ها
1,458
لایک ها
325
امتیاز
83
پاسخ : سوالی از ترکیبیات


 

Olympiad

New Member
ارسال ها
1,268
لایک ها
134
امتیاز
0
سوال ترکیبیات

یک بازی یک نفره روی یک صفحه ی m*n با mn مهره که یک روی آنها سیاه و روی دیگر آنها سفید است ، انجام میشود. در شروع بازی کلیه ی مهره ها را روی صفحه طوری قرار میدهیم که هر مهره در یک خانه قرار گیرد و روی سفید همه ی مهره ها بالا باشد به جز در یکی از خانه های گوشه ای . در هر حرکت میتوانیم یک مهره که روی سیاه آن بالا است را از روی صفحه حذف کنیم و مهره ی کلیه ی خانه های مجاور ضلعی آن را پشت و رو کنیم . (m,n) هایی را بیابید که بتوان همه ی مهره ها را حذف کرد .
 

mimilad

New Member
ارسال ها
298
لایک ها
40
امتیاز
0
پاسخ : سوال ترکیبیات

واسه اثبات سوال نشان میدهیم که اگر یکی از m و n زوجیت فرد داشته باشند حکم برقار میشود و اگر هر دو زوجیت زوج داشته باشند این کار قابل انجام نیست .
اثبات قسمت اول که با استقرا خیلی راحته کافیه n رو ثابت بگیرید و روی m استقرا بزنید . واسه قسمت دوم هم زوجیت تعداد تغییر های پشت و رو شدن تمام مهره ها را به دو طریق میشماریم از یک طرف به ازای هر دوخانه ی مجاور
دقیقا یکی به خاطر دیگری پشت ورو میشود و چون تعداد زوج های مجاور زوج است پس تعداد پشت رو شدن ها زوج است و از طرف دیگر میدانیم تمام خانه های سفید (دارای مهره ی سفید) برای حذف شدن باید فرد بار پشت رو شوند و خانه ی سیاه گوشه نیز در زوج بار پشت و رو میشود ( پشت رو نمیشود) پس در کل فرد بار پشت و رو میشوند که این تناقض است
 

Olympiad

New Member
ارسال ها
1,268
لایک ها
134
امتیاز
0
سوال ترکیبیات (Canada 2010)

هر راس از یک گراف متناهی میتوانند به یکی از 2 رنگ سفید یا سیاه رنگ آمیزی شوند . در ابتدا رنگ همه ی راس ها سیاه می باشند . در هر مرحله میتوانیم راسی مانند P را انتخاب کنیم و رنگ راس p و همه ی همسایه های P را تغییر دهیم . آیا میتوان رنگ همه ی راس ها را طی چند مرحله از سیاه به سفید تبدیل کنیم ؟
 
لایک ها SABB

SABB

New Member
ارسال ها
704
لایک ها
25
امتیاز
0
پاسخ : سوال ترکیبیات (Canada 2010)

از استقرا روی تعداد یالها استفاده میکنیم تا ثابت کنیم از هر گرافی میشه به یه گراف رسید که رنگ همه ی راس هاش برعکس گراف اولیه است. (e=0 بدیهیه).
اول از همه برای هر راس با درجه بزرگتر از صفر مثل v میدونیم G-v میتونه با یه روشی مثل (f(v به گرافی تبدیل شه که رنگ همه ی راس ها به جز v تغییر کرده، اگر بعد از این روش رنگ v هم تغییر کرد مساله حله. پس فرض کنیم به ازای هر v روش (f(v رنگ همه ی راس ها به جز v رو تغییر میده.
حالا یه یال e=uv رو میگیریم. میدونیم G-e رو میشه با یه روش رنگ همه ی راس هاش رو تغییر داد. اگه همون روش رو تو G پیاده کنیم، رنگ همه ی راس ها به جز u و v تغییر کرده و خود u و v‌ هم 2 حالت دارن: یا رنگ هر دوشون تغییر کرده که مساله حله، یا رنگ هیچکدوم تغییر نکرده. حالا اگه روش (f(u و روش (f(v رو انجام بدیم مساله حل میشه.
 

Olympiad

New Member
ارسال ها
1,268
لایک ها
134
امتیاز
0
سوال ترکیبیات

یک n ضلعی محدب و m مهره داریم که m>n میباشد . در هر مرحله میتونیم 2 مهره از یک راس برداریم و یکی رو به همسایه ی راستی و اون یکی رو به همسایه چپی بفرستیم !!!!! ثابت کنید اگر بعد از تعدادی مرحله به حالت اولیه برگردیم تعداد حرکات مضربی از n بوده است .
 

mimilad

New Member
ارسال ها
298
لایک ها
40
امتیاز
0
پاسخ : سوال ترکیبیات

اکسترمال بزنید و راسی راکه بیشترین حرکت را داده در نظر بگیرید و ثابت کنید دو راس اطرافش هم k حرکت انجام داده اند .
 

nima tn

New Member
ارسال ها
150
لایک ها
25
امتیاز
0
پاسخ : ماراتن ترکیبیات

سلام يك جدول. ٦ *٦ داريم با آبى و. قرمز. ميخوايم. رنگش كنيم. چند حالت. داره. كه. در. هر. سطر. و. ستون. ٢.تا. مربع. قرمز. باشه؟؟؟؟؟؟
 

Ar@sh

Moderator
ارسال ها
267
لایک ها
530
امتیاز
93
پاسخ : ماراتن ترکیبیات

موضوع مهم شد

و تمامی تاپیک هایی که عنوان "سوال ترکیبیات" داشتند به این تاپیک افزوده شدند :53:
 
آخرین ویرایش توسط مدیر

crazyboy

New Member
ارسال ها
413
لایک ها
539
امتیاز
0
پاسخ : ماراتن ترکیبیات

سلام يك جدول. ٦ *٦ داريم با آبى و. قرمز. ميخوايم. رنگش كنيم. چند حالت. داره. كه. در. هر. سطر. و. ستون. ٢.تا. مربع. قرمز. باشه؟؟؟؟؟؟
5!6!/2
توی هر ستون یدونه قرمز بزار که باهم تو ردیف تداخل نکنند (یعنی تو هر ردیف یدون قرمز و تو هر ستون یه قرمز) بعدش تو هر ستون تک تک یک خونه انتخاب کن :4:
حالتارو دوبار شمردیم تقسیم به دو میکنیم !
 
آخرین ویرایش توسط مدیر

bgo

New Member
ارسال ها
276
لایک ها
397
امتیاز
0
پاسخ : ماراتن ترکیبیات

اینم یه سوال ساده صرفن جهت راه افتادن ماراتن:
k مهره در جدول n*n داریم میدانیم اگر در خانه (i,j) مهره نباشد حداقل n مهره در سطر iام و ستون jام (مجموعن) قرار دارد ثابت کنید:
 
آخرین ویرایش توسط مدیر
بالا